All Questions

Filter by
Sorted by
Tagged with
15 votes
2 answers
696 views

Subsets of $(\mathbb{Z}/p)^{\times n}$

There seems to be some combinatorial fact that every subset $A$ of $G=(\mathbb{Z}/p)^{\times n}$ of cardinality $\frac{p^n-1}{p-1}+1$ containing $\vec{0}$ satisfies $(p-1)A=G$. ($p$ is a prime number....
Adam Chapman's user avatar
9 votes
0 answers
262 views

If $A+A+A$ contains the extremes, does it contain the middle?

Let $b \ge 1$ and $A\subseteq [0,b]$ be a set of integers (all intervals will be of integers). Write $hA := \underbrace{A + \ldots + A}_{h\text{ summands}} = \{ \sum_{i=1}^h a_i ~|~a_i \in A,\, \...
Alufat's user avatar
  • 825
3 votes
2 answers
333 views

Sumsets with the property "$A+B=C$ implies $A=C-B$"

Let $(G,+)$ be an abelian group and $A$, $B$ and $C$ be finite subsets of $G$ with $A+B=C$. One may conclude that $A\subset C-B$. However, $A$ need not be equal to $C-B$. What is a necessary and ...
Shahab's user avatar
  • 421
7 votes
1 answer
192 views

Trisecting $3$-fold sumsets, II: is the middle part ever thin?

This is a refined version of the question I asked yesterday. Let $A$ be a finite set of integers with the smallest element $0$ and the largest element $l$. The sumset $C:=3A$ resides in the interval $[...
Seva's user avatar
  • 22.6k
6 votes
1 answer
154 views

Trisecting $3$-fold sumsets: is the middle part always thick?

Here is a truly minimalistic and seemingly basic question which should have a simple solution (I hope it does). Let $A$ be a finite set of integers with the smallest element $0$ and the largest ...
Seva's user avatar
  • 22.6k
3 votes
1 answer
324 views

Prime gap distribution in residue classes and Goldbach-type conjectures

Update on 7/20/2020: It appears that conjecture A is not correct, you need more conditions for it to be true. See here (an answer to a previous MO question). The general problem that I try to solve is ...
Vincent Granville's user avatar
0 votes
1 answer
439 views

Congruential equidistribution, prime numbers, and Goldbach conjecture

Let $S$ be an infinite set of positive integers, $N_S(z)$ be the number of elements of $S$ less than or equal to $z$, and let $$D_S(z, n, p)= \sum_{k\in S,k\leq z}\chi(k\equiv p\bmod{n}).$$ Here $\chi$...
Vincent Granville's user avatar
0 votes
0 answers
152 views

General asymptotic result in additive combinatorics (sums of sets)

Let $S_1,\cdots,S_k$ be $k$ infinite sets of positive integers. Let $N_i(z)$ be the numbers of elements in $S_i$ that are less or equal to $z$. Let us further assume that $$N_i(S) \sim \frac{a_i z^{...
Vincent Granville's user avatar
5 votes
1 answer
152 views

Computational version of inverse sumset question

Let $p$ be prime and $\mathbb{F}_p$ the finite field with $p$ elements. Suppose we have a set $B\subseteq \mathbb{F}_p$ satisfying $|B|<p^{\alpha}$ for some $0<\alpha<1$ and there exists $A\...
user avatar
1 vote
1 answer
205 views

Average size of iterated sumset modulo $p-1$,

Given a prime $p$, what is the average size of the iterated sumset, $|kA|$, modulo $p-1$, with $p$ a prime, and $k$ given, with $A$ chosen at random? You can pick any type of prime you like for $p$, ...
Matt Groff's user avatar
19 votes
4 answers
851 views

Size of sets with complete double

Let $[n]$ denote the set $\{0,1,...,n\}$. A subset $S\subseteq [n]$ is said to have complete double if $S+S=[2n]$. Let $m(n)$ be the smallest size of a subset of $[n]$ with complete double. My ...
Hailong Dao's user avatar
  • 30.2k
3 votes
1 answer
218 views

On particular sumset properties of permanent?

Denote $\mathcal R_2[n]=\mathcal R[n] + \mathcal R[n]$ to be sumset of integers in $\mathcal R[n]$ where $\mathcal R[n]$ to be set of permanents possible with permanents of $n\times n$ matrices with $...
Turbo's user avatar
  • 13.5k
3 votes
1 answer
247 views

Are there unique additive decompositions of the reals?

Given $b\in \mathbb{R}_{>1}$ is there $U\subseteq\mathbb{R}_{\ge 0}$ such that $U+bU=\mathbb{R}_{\ge 0}$ and $(U-U)\cap b(U-U)=\{0\}$ (or equivalently: $u+bv=u'+bv' \implies u=u', v=v'$)? Here is ...
Yaakov Baruch's user avatar
4 votes
0 answers
149 views

Dividing a finite arithmetic progression into two sets of same sum: always the same asymptotics?

This is inspired by the recent question How many solutions $\pm1\pm2\pm3…\pm n=0$. The oeis entries A063865 linked to this question and A292476/A156700 for the related one "How many solutions $\pm1\...
Wolfgang's user avatar
  • 13.1k
2 votes
0 answers
137 views

The set of lengths of $nX$ gets larger and larger for every non-zero, non-empty, finite $X \subseteq \mathbf N$ with $0 \in X$

Let $H$ be a multiplicatively written monoid with identity $1_H$. Given $x \in H$, we take ${\sf L}_H(x) := \{0\}$ if $x = 1_H$; otherwise, ${\sf L}_H(x)$ is the set of all $k \in \mathbf N^+$ for ...
Salvo Tringali's user avatar
10 votes
1 answer
537 views

what is the status of this problem? an equivalent formulation?

R. Guy, Unsolved problems in number theory, 3rd edition, Springer, 2004. In this book, on page 167-168, Problem C5, Sums determining members of a set, discusses a question Leo Moser asked: suppose $X\...
T. Amdeberhan's user avatar
4 votes
2 answers
402 views

How big must the sumset $A+A$ be if $A$ satisfies no translation-invariant equations of low height?

Suppose $A$ is a finite subset of an abelian group. If there is no solution to $ma+nb=(m+n)c$ with $0\leq m,n\leq M$, can we bound $|A+A|$ from below? I am interested if one can obtain bounds much ...
deadcat's user avatar
  • 41
1 vote
1 answer
179 views

Sumset achieving extreme upper bound [closed]

It is trivial that $|A_1 + \cdots + A_h| \leq |A_1|\cdots |A_h|$, where $h \geq 2$ and $A_i \subseteq \mathbb{Z}$ are nonempty finite sets and $A_1 + \cdots + A_h :=\{a_1 + \cdots + a_h : a_i \in A_i ~...
Rajkumar's user avatar
  • 167
3 votes
2 answers
408 views

Sumsets with distinct numbers, upper bound for maximum element

Let $A$ be a finite set of positive natural numbers with $n$ elements, $|A|=n$, with the property that all sums of two (not necessarily different) elements are distinct, or in the usual notation for ...
Doc Brown's user avatar
  • 133
3 votes
3 answers
722 views

Is the sumset or the sumset of the square set always large?

Let A be a finite subset of $\mathbb{N}$, $\mathbb{R}$, or a sufficiently small subset of $\mathbb{F}_{p}$. Do we have a lower bound of the form $|A|^{1+\delta}$ on the following quantity: $$\max (|\...
Mark Lewko's user avatar
  • 11.7k
10 votes
2 answers
622 views

Sumsets and dilates: does $|A+\lambda A|<|A+A|$ ever hold?

The following problem is somehow hidden in this recently asked question, but I believe that it deserves to be asked explicitly. Is it true that for any finite set $A$ of real numbers, and any real $...
Seva's user avatar
  • 22.6k
4 votes
0 answers
214 views

Subgroup cliques in the Paley graph

It is a famous open problem to estimate non-trivially, for a prime $p\equiv 1\pmod 4$, the largest size of a subset $A\subset{\mathbb F}_p$ such that the difference of any two elements of $A$ is a ...
Seva's user avatar
  • 22.6k
3 votes
3 answers
469 views

How to find an integer set, s.t. the sums of at most 3 elements are all distinct?

How to find a set $A \subset \mathbb{N}$ such that any sum of at most three Elements $a_i \in A$ is different if at least one element in the sum is different. Example with $|A|=3$: Out of the set $A :...
Shannon's user avatar
  • 71
4 votes
1 answer
146 views

$B_k[1]$ sets with smallest possible $m = \max B_k[1]$ for given $k$ and $n = \lvert B_k[1]\rvert$ elements

Sidon sets are sets $A \subset \mathbb{N}$ such that for all $a_j,b_j \in A$ holds $$a_1+a_2=b_1+b_2 \iff \{a_1,a_2\}=\{b_1,b_2\}.$$ Thus if you know the sum of two elements, you know which elements ...
Shannon's user avatar
  • 71
26 votes
1 answer
771 views

Distribution of $a^2+\alpha b^2$

It is well known that size of the set of positive integers up to $n$ that can be written as $a^2+b^2$ is asymptotic to $C \frac{n}{\sqrt{\log n}}$. Here I'm interested mostly in the weaker fact that ...
Rodrigo's user avatar
  • 1,225
11 votes
1 answer
600 views

Lower bounds for $|A+A|$ if $A$ contains only perfect squares

Let $A$ a set with $|A|=n$ that contains only perfect squares of integers. What lower bounds can we give for $|A+A|$? I think the lower bound $\gg \frac{n^2}{\sqrt{log \,n}}$ holds (this would be ...
Rodrigo's user avatar
  • 1,225
5 votes
1 answer
428 views

A problem related with 'Postage stamp problem'

A friend of mine taught me this question. I found that it is related with 'Postage stamp problem' (though it does not seem to be same). Let $m,a_1\lt a_2\lt \cdots\lt a_n$ be natural numbers. Now let ...
mathlove's user avatar
  • 4,697
1 vote
1 answer
196 views

unique sums in a finite direct product of sets of integers

I am an algebraist, and I am wondering if there is a definition for the following: Let $A_1$, $A_2$, $\ldots, A_n$ be sets of integers (or more generally, subsets of a group $G$). Say that (for the ...
Greg's user avatar
  • 11
11 votes
0 answers
812 views

Cliques in the Paley graph and a problem of Sarkozy

The following question is motivated by pure curiosity; it is not a part of any research project and I do not have any applications. The question comes as an interpolation between two notoriously ...
Seva's user avatar
  • 22.6k